강하향 반사슬 문서 원본 보기
←
강하향 반사슬
둘러보기로 이동
검색으로 이동
문서 편집 권한이 없습니다. 다음 이유를 확인해주세요:
요청한 명령은 다음 권한을 가진 사용자에게 제한됩니다:
사용자
.
문서의 원본을 보거나 복사할 수 있습니다.
{{위키데이터 속성 추적}} [[순서론]]에서 '''강하향 반사슬'''(強下向反사슬, {{llang|en|strong downward antichain}})은 서로 다른 두 원소가 공통된 [[하계 (수학)|하계]]를 갖지 않는, [[원순서 집합]]의 [[반사슬]]이다. 반대로, '''강상향 반사슬'''(強下向反사슬, {{llang|en|strong downward antichain}})은 서로 다른 두 원소가 공통된 [[상계 (수학)|상계]]를 갖지 않는, [[원순서 집합]]의 [[반사슬]]이다. == 정의 == === 원소의 양립 === [[원순서 집합]] <math>(X,\lesssim)</math>의 두 원소 <math>x,y\in X</math>에 대하여, 만약 <math>\{x,y\}</math>가 [[하계 (수학)|하계]]를 갖는다면, <math>x</math>와 <math>y</math>가 '''하향 양립'''(下向兩立, {{llang|en|downward compatible}})한다고 한다.<ref name="Kunen"/>{{rp|53, Definition II.2.2}} 마찬가지로, 만약 <math>\{x,y\}</math>가 [[하계 (수학)|하계]]를 갖는다면, <math>x</math>와 <math>y</math>가 '''상향 양립'''(上向兩立, {{llang|en|upward compatible}})한다고 한다. 서로 비양립하는 두 원소는 보통 <math>x\perp y</math>로 표기하며, 서로 양립하는 두 원소는 보통 <math>x\parallel y</math>로 표기한다. (이 기호들이 상향·하향을 나타내는지 여부는 문헌에 따라 다르다.) === 강하향·강상향 반사슬 === [[원순서 집합]] <math>(X,\lesssim)</math>에 대하여 다음 두 조건이 서로 [[동치]]이며, 이를 만족시키는 [[원순서 집합]]을 '''강하향 반사슬'''이라고 한다.<ref name="Kunen">{{서적 인용|title=Set theory: an introduction to independence proofs|성=Kunen|이름=Kenneth|저자링크=케네스 쿠넌|publisher=North-Holland|날짜=1980|isbn=978-0-444-86839-8|url=http://store.elsevier.com/Set-Theory-An-Introduction-To-Independence-Proofs/K_-Kunen/isbn-9780444868398/|총서=Studies in Logic and the Foundations of Mathematics|권=102|zbl=0534.03026|mr=597342|언어=en|확인날짜=2016-08-11|보존url=https://web.archive.org/web/20160911102401/http://store.elsevier.com/Set-Theory-An-Introduction-To-Independence-Proofs/K_-Kunen/isbn-9780444868398/|보존날짜=2016-09-11|url-status=dead}}</ref>{{rp|53, Definition III.2.2}} * <math>A</math>의 [[부분 집합]] 가운데 크기 2 이상인 것은 [[하계 (수학)|하계]]를 갖지 않는다. * 임의의 <math>a,a'\in A</math>에 대하여, 만약 <math>a</math>와 <math>a'</math>이 하향 양립한다면, <math>a=a'</math>이다. * 임의의 <math>a,a'\in A</math> 및 <math>x\in X</math>에 대하여, <math>x\lesssim a\ne a'</math>라면 <math>x\not\lesssim a'</math>이다. 마찬가지로, [[원순서 집합]] <math>(P,\lesssim)</math>에 대하여 다음 두 조건이 서로 [[동치]]이며, 이를 만족시키는 [[원순서 집합]]을 '''강하향 반사슬'''이라고 한다. * <math>A</math>의 [[부분 집합]] 가운데 크기 2 이상인 것은 [[상계 (수학)|상계]]를 갖지 않는다. * 임의의 <math>a,a'\in A</math>에 대하여, 만약 <math>a</math>와 <math>a'</math>이 상향 양립한다면, <math>a=a'</math>이다. * 임의의 <math>a,a'\in A</math> 및 <math>x\in X</math>에 대하여, <math>x\gtrsim a\ne a'</math>라면 <math>x\not\gtrsim a'</math>이다. 강한 하향 반사슬 및 강한 상향 반사슬은 [[반사슬]]이지만, 그 역은 성립하지 않는다. === 반사슬 조건 === 임의의 [[기수 (수학)|기수]] <math>\kappa</math>가 주어졌다고 하자. 만약 [[원순서 집합]] <math>(X,\lesssim)</math>의 모든 강하향 반사슬의 크기가 <math>\kappa</math> 미만이라면, <math>(X,\lesssim)</math>가 '''<math>\kappa</math>-강하향 반사슬 조건'''(<math>\kappa</math>-強上向反사슬條件, {{llang|en|<math>\kappa</math>-strong downward antichain condition}})을 만족시킨다고 한다.<ref name="Jech">{{서적 인용 | last=Jech | first=Thomas | title=Set theory | url=https://archive.org/details/settheory0000jech_f7i4 | publisher= Springer-Verlag | series=Springer Monographs in Mathematics | 날짜=2003 | doi=10.1007/3-540-44761-X | issn= 1439-7382 | 판 = 3 | isbn= 978-3-540-44085-7 | zbl = 1007.03002 | 언어=en | id={{iaid|settheory0000jech_f7i4}} }}</ref>{{rp|227, Definition 15.2}}<ref name="Kunen"/>{{rp|212, Definition VII.6.7}} 마찬가지로, [[원순서 집합]] <math>(X,\lesssim)</math>의 모든 강상향 반사슬의 크기가 <math>\kappa</math> 미만이라면, <math>(P,\lesssim)</math>가 '''<math>\kappa</math>-강상향 반사슬 조건'''(<math>\kappa</math>-強上向反사슬條件, {{llang|en|<math>\kappa</math>-strong upward antichain condition}})을 만족시킨다고 한다. 특히, 만약 <math>\kappa=\aleph_1</math>일 경우, 이 조건들은 '''가산 강하향 반사슬 조건'''(可算強上向反사슬條件, {{llang|en|countable strong downward antichain condition}})·'''가산 강상향 반사슬 조건'''(可算強上向反사슬條件, {{llang|en|countable strong downward antichain condition}})이라고 한다.<ref name="Jech"/>{{rp|220, Definition 14.33}}<ref name="Kunen"/>{{rp|53, Definition II.2.3}}<ref name="Schindler">{{서적 인용|제목=Set theory: exploring independence and truth|이름=Ralf|성=Schindler|doi=10.1007/978-3-319-06725-4|출판사=Springer-Verlag|총서=Universitext|날짜=2014|issn=0172-5939|isbn=978-3-319-06724-7|언어=en}}</ref> (집합론에서는 이 조건들을 보통 "가산 반사슬 조건" 또는 "가산 사슬 조건"으로 부른다. 후자는 [[불 대수]]의 경우 사슬과 반사슬 사이의 관계에 따른 것이다.) == 성질 == <math>B</math>가 [[완비 불 대수]]라고 하고, <math>\kappa</math>가 임의의 [[기수 (수학)|기수]]라고 하자. 그렇다면, 다음 두 집합 사이에는 자연스러운 [[전단사 함수]]가 존재한다. * 그 [[상 (수학)|상]]이 <math>B\setminus\{\bot\}</math>의 강하향 반사슬을 이루는 함수 <math>f\colon\kappa\to B\setminus\{\bot\}</math>들의 집합 * [[단사 함수|단사]] [[증가 함수]] <math>g\colon \kappa\to B</math> 가운데, 임의의 <math>S\subseteq\kappa</math>에 대하여 (<math>\sup S<\kappa</math>), <math>\sup g(S)=g(\sup S)</math>인 것들의 집합. (특히, <math>S=\varnothing</math>일 때 <math>\bot=g(0)</math>이다.) 구체적으로, 이 [[전단사 함수]]는 다음과 같다. <math>f\colon \kappa\to B\setminus\{\bot\}</math>가 주어졌을 때, :<math>g\colon\alpha\mapsto\sup_{\beta<\alpha}f(\beta)\in B</math> 를 정의한다. 반대로, <math>g\colon\kappa\to B</math>가 주어졌을 때, :<math>f\colon\alpha\mapsto g(\alpha+1)\land\lnot g(\alpha)</math> 를 정의한다. == 예 == [[공집합]]과 [[한원소 집합]]은 (자명하게) 항상 강하향 반사슬이자 강상향 반사슬이다. [[최대 원소]]를 갖는 [[원순서 집합]]의 강상향 반사슬은 [[공집합]]이거나 [[한원소 집합]]이다. 마찬가지로, [[최소 원소]]를 갖는 [[원순서 집합]]의 강하향 반사슬은 [[공집합]]이거나 [[한원소 집합]]이다. === 멱집합 === 집합 <math>X</math>에 대하여, [[부분 순서 집합]] <math>(\mathcal P(X)\setminus\{\varnothing\},\subseteq)</math>을 생각하자. 이 경우, [[공집합]]을 포함하지 않는 집합족 <math>\mathcal S\subseteq \mathcal P(X)\setminus\{\varnothing\}</math>에 대하여 다음 두 조건이 서로 [[동치]]이다. * <math>\mathcal P(X)\setminus\{\varnothing\}</math>의 강하향 반사슬이다. * [[서로소 집합|서로소 집합족]]이다. 즉, 임의의 두 <math>A,B\in\mathcal S</math>에 대하여, <math>A\ne B</math>라면 <math>A\cap B=\varnothing</math>이다. 특히, <math>\mathcal P(X)\setminus\{\varnothing\}</math>의 강하향 반사슬의 최대 크기는 <math>|X|</math>이다. 마찬가지로, 집합족 <math>\mathcal S\subseteq \mathcal P(X)\setminus\{X\}</math>에 대하여 다음 세 조건이 서로 [[동치]]이다. * <math>\mathcal P(X)\setminus\{X\}</math>의 강상향 반사슬이다. * 임의의 두 <math>A,B\in\mathcal S</math>에 대하여, <math>A\cup B=X</math>이다. * <math>\{X\setminus S\colon S\in\mathcal S\}</math>는 [[서로소 집합|서로소 집합족]]이다. 특히, <math>\mathcal P(X)\setminus\{X\}</math>의 강상향 반사슬의 최대 크기는 <math>|X|</math>이다. <math>\mathcal P(X)\setminus\{\varnothing,X\}</math> 속의 반사슬 가운데, 반하향 반사슬이자 반상향 반사슬인 것들을 생각한다면, 이는 항상 크기가 2 이하이다. 즉, (크기 2일 경우) <math>\varnothing\ne S\subsetneq X</math>에 대하여 <math>\{S,X\setminus S\}</math>의 꼴이다. === 위상 공간 === [[위상 공간 (수학)|위상 공간]] <math>X</math>의, [[공집합]]이 아닌 [[열린집합]]들의 [[부분 순서 집합]] <math>\left(\operatorname{Open}(X)\setminus\{\varnothing\},\subseteq\right)</math>을 생각하자. 만약 <math>X</math>가 [[수슬린 가설|수슬린 직선]]이라면, <math>\operatorname{Open}(X)\setminus\{\varnothing\}</math>는 (정의에 따라) 가산 강하향 반사슬 조건을 만족시킨다. 만약 <math>X</math>가 [[분해 가능 공간]]이라면, <math>\operatorname{Open}(X)\setminus\{\varnothing\}</math>는 가산 강하향 반사슬 조건을 만족시킨다. 사실, [[거리화 가능 공간]]에 대하여 다음 두 조건이 서로 [[동치]]이다. * [[분해 가능 공간]]이다. * 가산 강하향 반사슬 조건이 성립한다. 집합 <math>\{0,1\}^{\beth_2}</math>에 [[곱공간]] 위상을 줄 경우, 이는 가산 강하향 반사슬 조건을 만족시키지만, 이는 [[분해 가능 공간]]이 아니다. (여기서 <math>\beth_2=2^{2^{\aleph_0}}</math>는 [[베트 수]]이다.) == 응용 == 강하향·강상향 반사슬의 개념은 [[강제법]]에 등장한다. 강제법에서, 만약 [[공시작 집합]]과 [[포괄적 필터]]를 사용할 경우 가산 강하향 반사슬 조건을 사용하며, 반대로 [[공종 집합]]과 [[포괄적 순서 아이디얼]]을 사용할 경우 가산 강상향 반사슬 조건을 사용한다. == 각주 == {{각주}} == 외부 링크 == * {{eom|title=Chain condition}} * {{nlab|id=countable chain condition|title=Countable chain condition}} * {{웹 인용|url=http://mathoverflow.net/questions/47057/does-antichain-mean-something-different-in-set-forcing-than-in-lattice-theory|제목=Does “antichain” mean something different in set-forcing than in lattice theory?|출판사=Math Overflow|언어=en}} {{전거 통제}} [[분류:순서론]]
이 문서에서 사용한 틀:
틀:Eom
(
원본 보기
)
틀:Llang
(
원본 보기
)
틀:Nlab
(
원본 보기
)
틀:Rp
(
원본 보기
)
틀:각주
(
원본 보기
)
틀:서적 인용
(
원본 보기
)
틀:웹 인용
(
원본 보기
)
틀:위키데이터 속성 추적
(
원본 보기
)
틀:전거 통제
(
원본 보기
)
강하향 반사슬
문서로 돌아갑니다.
둘러보기 메뉴
개인 도구
로그인
이름공간
문서
토론
한국어
보기
읽기
원본 보기
역사 보기
더 보기
검색
둘러보기
대문
최근 바뀜
임의의 문서로
미디어위키 도움말
특수 문서 목록
도구
여기를 가리키는 문서
가리키는 글의 최근 바뀜
문서 정보